Jump to content

L'Hôpital's rule: Difference between revisions

From Wikipedia, the free encyclopedia
Content deleted Content added
STBot (talk | contribs)
m clean up using AWB
Added and organized proofy things.
(One intermediate revision by the same user not shown)
Line 161: Line 161:
|}
|}


==Proofs of L'Hôpital's rule==
==Proof==
==General proof by definition of derivative==


:<math>
The most common proof of l'Hôpital's rule uses [[mean value theorem|Cauchy's mean value theorem]].
\lim_{x\to c}{f(x) \over g(x)} = \lim_{e\to 0}{f(c+e) \over g(c+e)}
</math>

Or, in other words, when ''e'' is infinitely small, then

:<math>
{f(c+e) \over g(c+e)}
= {f(c) \over g(c)},
</math>

using ''a''/''b'' = ''c''/''d'' = (''a''&nbsp;&minus;&nbsp;''c'')/(''b''&nbsp;&minus;&nbsp;''d'') we can write, for ''e'' approaching 0,

:<math>
{f(x) \over g(x)} = {[f(c+e)-f(c)] \over [g(c+e)-g(c)]}.
</math>

Dividing both numerator and denominator by ''e''

:<math>
{f(x) \over g(x)}~= {[f(c+e)-f(c)]/e \over [g(c+e)-g(c)]/e},
</math>

taking the limit when ''e'' approaches 0 we observe that the numerator and denominator on the right hand side of the equation are actually ''f''&prime;(''c'') and ''g''&prime;(''c'') respectively, therefore

:<math>
\lim_{x\to c}{f(x) \over g(x)}
= {f'(c) \over g'(c)}.
</math>

The proof is independent of whether <math>\lim_{x \to c}f(x) = \lim_{x \to c} g(x) = 0</math> or <math>\lim_{x \to c}|g(x)|=\infty</math>

===Proof by Cauchy's mean value theorem===
The most common proof of L'Hôpital's rule uses [[mean value theorem|Cauchy's mean value theorem]].


1) The case when <math>f(x) \to 0, g(x) \to 0</math>
1) The case when <math>f(x) \to 0, g(x) \to 0</math>
Line 216: Line 250:
we show that the limit of ''f''(''x'')/''g''(''x'') tends to the same when <math>x \to c</math> and <math>h \to 0</math>.
we show that the limit of ''f''(''x'')/''g''(''x'') tends to the same when <math>x \to c</math> and <math>h \to 0</math>.


=== Other proofs ===
===Proof by local linearity===
Suppose that functions <math>f(x)</math> and <math>g(x)</math> are [[continuous]] and [[Derivative#Differentiation and differentiability|differentiable]] on the [[Interval (mathematics)| interval]] <math>(a, b)</math>. In addition, there is a function <math>h(x)</math> that is equal to the ratio of <math>f(x)</math> and <math>g(x)</math>. In other words,
:<math>h(x)={{f(x)}\over{g(x)}}</math>
When these functions are equal to a [[real number]] <math>c</math>, which is on the interval <math>(a, b)</math>, their ratio
:<math>h(c)={{f(c)}\over{g(c)}}</math>
For example, if <math>f(x)=x^3+1</math> and <math>g(x) = \sin \left(\frac{\pi x}{4}\right)</math>, then <math>h(x)</math> evaluated at 1 is
:<math>h(1)={{f(1)}\over{g(1)}}={{1^3+1}\over{\sin \left(\frac{\pi \cdot 1}{4}\right)}}={{2}\over{1 / \sqrt{2}}}=2 \sqrt{2}</math>
However, suppose that <math>f(c)=g(c)=0 \,</math>, where <math>c</math> is again a value on the interval <math>(a, b)</math>. Then
:<math>h(c)={{f(c)}\over{g(c)}}={0 \over 0}</math>
0 divided by 0 is [[Indeterminate (variable)|indeterminate]]. This means that <math>h(c)</math> could be any real number or infinity, based on the context. However, <math>f(x)</math> and <math>g(x)</math> are continuous and differentiable at <math>x = c</math>, they can be approximate by lines (because of [[local linearity]].


The [[linear approximation]] (or [[linearization]]) of <math>f(x)</math> and <math>g(x)</math> around <math>x = c</math> is, respectively, <math>y=f(c)+f'(c)\,(x-c)</math> and <math>y=g(c)+g'(c)\,(x-c)</math>. The function <math>h(x)</math> near <math>x=c</math>, then, can be approximated by
There are more intuitive proofs of the rule. If
:<math>\lim_{x \to a} \frac{f(x)}{g(x)}</math>
:<math>{\lim_{x\rightarrow c}} \ h(c)={\lim_{x\rightarrow c}{{\mbox{Linearization of } f\left(x\right)}\over{\mbox{Linearization of } g\left(x\right)}}}</math>
Using the mathematical expression for the functions' linearizations,
tends to the indeterminate form 0/0, then the rule can be proven with a [[local linearity]] argument. If it tends to the indeterminate form <math>{\infty}/{\infty}</math>, then this can be converted to 0/0 form using the identity :<math>\frac{f(x)}{g(x)}=\frac{1/g(x)}{1/f(x)}.</math>
:<math>{\lim_{x\rightarrow c}} \ h(c)={\lim_{x\rightarrow c}{{f\left(c\right)+f'\left(c\right)\,\left(x-c\right)} \over{g\left(c\right)+g'\left(c\right)\,\left(x-c\right)}}}</math>
By assuming this limit equals ''L'', and taking the derivative of the numerator and denominator, it can be proven that <math>L=\lim_{x \to a}\frac{f'(x)}{g'(x)}</math>.
Since <math>f(c)</math> and <math>g(c)</math> and 0 regardless of the value of <math>x</math>,
:<math>{\lim_{x\rightarrow c}} \ h(c)={\lim_{x\rightarrow c}{{f'\left(c\right)\,\left(x-c\right)} \over{g'\left(c\right)\,\left(x-c\right)}}}</math>
There is factor of <math>(x - c)</math> in both the numerator and denominator, which can be canceled, revealing
:<math>{\lim_{x\rightarrow c}} \ h(c)={\lim_{x\rightarrow c}{{f'\left(c\right)} \over{g'\left(c\right)}}}</math>
If <math>f'(c)</math> and <math>g'(c)</math> are both real and <math>g'(c)</math> is not zero, then
:<math>{\lim_{x\rightarrow c}}\ h(c)={\lim_{x\rightarrow c}} \ {{f(c)} \over{g(c)}}={{f'(c)} \over{g'(c)}}</math>
If <math>g'(c)</math> approaches 0 but <math>f'(c)</math> does not, then then <math>h(c)</math> does not exist; or more specifically, it is either <math>\infty</math> or <math>-\infty</math> or approaches each from different sides.


If a 0 over 0 form occurs again, L'Hôpital's rule can be used again (unless <math>f(x)</math> and <math>g(x)</math> resemble the [[0 (number)#Extended use of zero in mathematics|zero function]] <math>y = 0</math> around <math>c</math>).
==General proof ==


This concludes the proof.
:<math>
\lim_{x\to c}{f(x) \over g(x)} = \lim_{e\to 0}{f(c+e) \over g(c+e)}
</math>


==Proof that L'Hôpital's can be applied to infinity over infinity==
Or, in other words, when ''e'' is infinitely small, then
The proof by definition of the derivative, the proof by Cauchy's mean value theorem, and the proof by local linearity all prove that
:<math>{\lim_{x\rightarrow c}} \ {{f(c)} \over{g(c)}}={\lim_{x\rightarrow c}} \ {{f'(c)} \over{g'(c)}}</math>
when <math>f(c)=g(c)=0</math>. Suppose that, instead of approaching zero, <math>f(c)=g(c)=\pm\infty</math>. In other words, for any <math>f(x)</math> or <math>g(x)</math> close to <math>x=c</math>, <math>f(x)</math> or <math>g(x)</math> gets either larger or smaller as you get closer to <math>x=c</math>, depending on whether the function goes to <math>\infty</math> or <math>-\infty</math> respectively. Only the proof by definition of derivative makes provisions for this circumstance. However, it is possible to use the proof for the 0 over 0 form have L'Hôpital's rule apply to <math>\infty</math> over <math>\infty</math>.


The <math>\infty</math> over <math>\infty</math> form (which is [[Indeterminate (variable)|indeterminate]]: it can mean any value) can be treated similarly when <math>f(c)=g(c)=\pm\infty</math>.
:<math>
:<math>{\lim_{x\rightarrow c}}\ \frac{f(x)}{g(x)}={\lim_{x\rightarrow c}}\ \frac{1/g(x)}{1/f(x)}</math>
{f(c+e) \over g(c+e)}
transforms an <math>\infty</math> over <math>\infty</math> form into a 0 over 0 form, and the latter form can be evaluated with L'Hôpital's rule. According to the rule (taking the derivative of the numerator and denominator,
= {f(c) \over g(c)},
:<math>{\lim_{x\rightarrow c}}\ \frac{1/g(x)}{1/f(x)} = {\lim_{x\rightarrow c}}\ \frac{-g'(x)/\left[g(x)\right]^2}{-f'(x)/\left[f(x)\right]^2}</math>
</math>
Simplifying the complex fractions,

:<math>{\lim_{x\rightarrow c}}\ \frac{-g'(x)/\left[g(x)\right]^2}{-f'(x)/\left[f(x)\right]^2} = {\lim_{x\rightarrow c}}\ \frac{g'(c) \left[f(x)\right]^2}{f'(c) \left[g(x)\right]^2}</math>
using ''a''/''b'' = ''c''/''d'' = (''a''&nbsp;&minus;&nbsp;''c'')/(''b''&nbsp;&minus;&nbsp;''d'') we can write, for ''e'' approaching 0,
Now, using the property of [[transitivity]] and the above statements,

:<math>{\lim_{x\rightarrow c}}\ \frac{f(x)}{g(x)} = {\lim_{x\rightarrow c}}\ \frac{g'(c) \left[f(x)\right]^2}{f'(c) \left[g(x)\right]^2}</math>
:<math>
Algebraically manipulating both sides, we find that
{f(x) \over g(x)} = {[f(c+e)-f(c)] \over [g(c+e)-g(c)]}.
:<math>{\lim_{x\rightarrow c}}\ \frac{f(x)}{g(x)} \cdot \frac{\left[g(x)\right]^2}{\left[f(x)\right]^2}= {\lim_{x\rightarrow c}}\ \frac{g'(c) \left[f(x)\right]^2}{f'(c) \left[g(x)\right]^2} \cdot \frac{\left[g(x)\right]^2}{\left[f(x)\right]^2}</math>
</math>
:<math>{\lim_{x\rightarrow c}}\ \frac{g(x)}{f(x)}= {\lim_{x\rightarrow c}}\ \frac{g'(c)}{f'(c)}</math>

And finally, when <math>f(c)=g(c)=\pm\infty</math>
Dividing both numerator and denominator by ''e''
:<math>{\lim_{x\rightarrow c}}\ \frac{f(x)}{g(x)}= {\lim_{x\rightarrow c}}\ \frac{f'(c)}{g'(c)}</math>

This is L'Hôpital's rule for the <math>\infty/\infty</math>. This concludes the proof.
:<math>
{f(x) \over g(x)}~= {[f(c+e)-f(c)]/e \over [g(c+e)-g(c)]/e},
</math>

taking the limit when ''e'' approaches 0 we observe that the numerator and denominator on the right hand side of the equation are actually ''f''&prime;(''c'') and ''g''&prime;(''c'') respectively, therefore

:<math>
\lim_{x\to c}{f(x) \over g(x)}
= {f'(c) \over g'(c)}.
</math>

The proof is independent of whether <math>\lim_{x \to c}f(x) = \lim_{x \to c} g(x) = 0</math> or <math>\lim_{x \to c}|g(x)|=\infty</math>


==Other applications==
==Other applications==

Revision as of 00:28, 30 October 2006

In calculus, l'Hôpital's rule (alternatively, l'Hospital's rule) uses derivatives to help compute limits with indeterminate forms. Application (or repeated application) of the rule often converts an indeterminate form to a determinate form, allowing easy computation of the limit. The rule is named after the 17th-century French mathematician Guillaume de l'Hôpital, who published the rule in his book Analyse des infiniment petits pour l'intelligence des lignes courbes (1696), the first book to be written on differential calculus.

The rule is believed to be the work of Johann Bernoulli. L'Hôpital, a nobleman, paid Bernoulli a retainer of 300 pounds per year to keep him updated on developments in calculus and to solve problems he had. Among these problems was that of limits of indeterminate forms. When l'Hôpital published his book, he gave due credit to Bernoulli and, not wishing to take credit for any of the mathematics in the book, he published the work anonymously. Bernoulli, who was known for being extremely jealous, claimed to be the author of the entire work, and until recently, it was believed to be so. Nevertheless, the rule was named for l'Hôpital, who never claimed to have invented it in the first place[1].

Overview

Introduction

In simple cases, L'Hôpital's rule states that for functions and :

if or then:

where the prime (') denotes the derivative.

Among other requirements, for this rule to hold, the limit must exist. Other requirements are detailed below, in the formal definition.

Formal statement

When determining the limit of a quotient when both f and g approach 0, or f and g approach infinity, L'Hôpital's rule states that has the same limit (if the limit exists), provided that g′ is nonzero throughout some interval containing the point in question. This differentiation often simplifies the quotient and/or converts it to a determinate form, allowing the limit to be determined more easily.

Symbolically let . Suppose that , that

and that for all in an open interval (a,b) containing c (or with if or with if ). If

then

L'Hôpital's rule also holds for one-sided limits.

Basic indeterminate forms (all others reduce to these):

Other indeterminate forms:

Note the requirement that the limit exists. Differentiation of limits of this form can sometimes lead to limits that do not exist. In that case, L'Hôpital's rule cannot be applied. For instance if and , then

does not exist, whereas

In practice one often uses the rule and, if the resulting limit exists, concludes that it was legitimate to use L'Hôpital's rule.

Note also the requirement that the derivative of g not vanish throughout an entire interval containing the point c. Without such a hypothesis, the conclusion is false. Thus one must not use L'Hôpital's rule if the denominator oscillates wildly near the point where one is trying to find the limit. For example if and , then

whereas

does not exist since fluctuates between e-1 and e.

Examples

  • Here is an example involving the sinc function, which has the form 0/0 :
However, it is simpler to observe that this limit is just the definition of the derivative of sin(x) at x = 0.
In fact this particular limit is needed in the most usual proof that the derivative of sin(x) is cos(x), but we cannot use l'Hôpital's rule to do this, as it would produce a circular argument.
  • Here is a more elaborate example involving the indeterminate form 0/0. Applying the rule a single time still results in an indeterminate form. In this case, the limit may be evaluated by applying l'Hôpital's rule three times:
  • Here is another case involving 0/0:
  • Here is a case of ∞/∞:
  • This one involves ∞/∞. Assume n is a positive integer.
Iterate the above until the exponent is 0. Then one sees that the limit is 0.
  • This one also involves ∞/∞:


  • And:

Proofs of L'Hôpital's rule

General proof by definition of derivative

Or, in other words, when e is infinitely small, then

using a/b = c/d = (a − c)/(b − d) we can write, for e approaching 0,

Dividing both numerator and denominator by e

taking the limit when e approaches 0 we observe that the numerator and denominator on the right hand side of the equation are actually f′(c) and g′(c) respectively, therefore

The proof is independent of whether or

Proof by Cauchy's mean value theorem

The most common proof of L'Hôpital's rule uses Cauchy's mean value theorem.

1) The case when

First, we expand continuously (or redefine) and by for . This doesn't change the limit since the limit doesn't depend on the value in the point (by definition).

According to Cauchy's mean value theorem there is a constant in such that:

Since ,

If then and

2) The case when

Let . Then using Cauchy's mean value theorem:

We rewrite that in the form

and then by the discussion of the two cases

we show that the limit of f(x)/g(x) tends to the same when and .

Proof by local linearity

Suppose that functions and are continuous and differentiable on the interval . In addition, there is a function that is equal to the ratio of and . In other words,

When these functions are equal to a real number , which is on the interval , their ratio

For example, if and , then evaluated at 1 is

However, suppose that , where is again a value on the interval . Then

0 divided by 0 is indeterminate. This means that could be any real number or infinity, based on the context. However, and are continuous and differentiable at , they can be approximate by lines (because of local linearity.

The linear approximation (or linearization) of and around is, respectively, and . The function near , then, can be approximated by

Using the mathematical expression for the functions' linearizations,

Since and and 0 regardless of the value of ,

There is factor of in both the numerator and denominator, which can be canceled, revealing

If and are both real and is not zero, then

If approaches 0 but does not, then then does not exist; or more specifically, it is either or or approaches each from different sides.

If a 0 over 0 form occurs again, L'Hôpital's rule can be used again (unless and resemble the zero function around ).

This concludes the proof.

Proof that L'Hôpital's can be applied to infinity over infinity

The proof by definition of the derivative, the proof by Cauchy's mean value theorem, and the proof by local linearity all prove that

when . Suppose that, instead of approaching zero, . In other words, for any or close to , or gets either larger or smaller as you get closer to , depending on whether the function goes to or respectively. Only the proof by definition of derivative makes provisions for this circumstance. However, it is possible to use the proof for the 0 over 0 form have L'Hôpital's rule apply to over .

The over form (which is indeterminate: it can mean any value) can be treated similarly when .

transforms an over form into a 0 over 0 form, and the latter form can be evaluated with L'Hôpital's rule. According to the rule (taking the derivative of the numerator and denominator,

Simplifying the complex fractions,

Now, using the property of transitivity and the above statements,

Algebraically manipulating both sides, we find that

And finally, when

This is L'Hôpital's rule for the . This concludes the proof.

Other applications

Many other indeterminate forms, such as , , and can be calculated using l'Hôpital's rule.

For example, to handle a case of , the difference of two functions is converted to a quotient:

The rule can be used on indeterminate forms involving exponents by using logarithms to "move the exponent down."

Other methods of computing limits

Although l'Hôpital's rule is a powerful way of computing otherwise hard-to-compute limits, it is not always the easiest. Some limits are actually easier to compute using the Taylor series expansion.

For example,

Some elementary algebraic manipulation, however, yields:

And applying l'Hôpital's rule, we have:

On the other hand, a simple substitution also allows the use of l'Hôpital's rule.

Therefore, as ,

Therefore,

Logical circularity

In some cases it may constitute circular reasoning to use l'Hôpital's rule to evaluate such limits as

If one uses the evaluation of the limit above for the purpose of proving that

and one uses l'Hôpital's rule and the fact that

in the evaluation of the limit, the argument uses the expected proof to prove itself and is therefore fallacious.

External links

References

  1. ^ Finney, Ross L. and George B. Thomas, Jr. Calculus. 2nd Edition. P. 390. Addison Wesley, 1994.